PT 28, Section 1, Question 23 Forum

Prepare for the LSAT or discuss it with others in this forum.
Post Reply
msw099

New
Posts: 22
Joined: Wed Nov 11, 2009 2:19 am

PT 28, Section 1, Question 23

Post by msw099 » Tue Feb 23, 2010 2:19 am

Hi All,

Can someone please explain how answer choice C weakens the argument. I narrowed down the choices to A and C and chose incorrectly.

Thank you in advance!

Shrimps

Bronze
Posts: 269
Joined: Sun Feb 14, 2010 10:04 pm

Re: PT 28, Section 1, Question 23

Post by Shrimps » Tue Feb 23, 2010 2:30 am

Hello again.

The argument: one's general disposition (incl. clinical depression, apparently) is the result of frontal lobe activity
Which answer would NOT weaken it?

(A) drugs that affect the frontal lobe change general disposition - does not weaken the argument (correct answer right here)
(B) asserts that the author got cause and effect wrong - it's the sleep (a symptom of depression) that affects frontal lobe activity, not the other way around - thus contradicting the argument
(C) asserts that there's no correlation between frontal lobe activity and disposition - thus contradicting the argument
(D) asserts that frontal lobe activity, again, is not causing but is CAUSED by something else in the brain - thus contradicting the argument
(E) same as B. Asserts that increased frontal lobe activity is a symptom, not the cause of a particular disposition.

User avatar
autarkh

Bronze
Posts: 314
Joined: Sat Sep 26, 2009 9:05 pm

Re: PT 28, Section 1, Question 23

Post by autarkh » Tue Feb 23, 2010 2:39 am

msw099 wrote:Hi All,

Can someone please explain how answer choice C weakens the argument. I narrowed down the choices to A and C and chose incorrectly.

Thank you in advance!

Look at the conclusion of the argument. They're claiming that disposition results from frontal lobe activity. "C" says that frontal lobe activity is not subject to variation in the same way as general disposition. That weakens the supposed causal connection between lobe activity and disposition.

"A" on the other hand says that anti-depressive drugs act by increasing left lobe activity. If you can counter depression by increasing activity, then it stands to reason that there is at least some link between activity and disposition. The argument is made stronger. Since this was a "weaken, EXCEPT", you have your answer.

msw099

New
Posts: 22
Joined: Wed Nov 11, 2009 2:19 am

Re: PT 28, Section 1, Question 23

Post by msw099 » Tue Feb 23, 2010 3:13 am

Thank you both!

Want to continue reading?

Register now to search topics and post comments!

Absolutely FREE!


Post Reply

Return to “LSAT Prep and Discussion Forum”